PT62 LR1 Q25
Posted: Wed Feb 06, 2013 9:31 pm
The credited response is (A) but I am SURE that this is incorrect. Here is my logic:
The question stem states that one possible explanation for the shrinking applicant pool is charging too little. It also states that to increase the size of the applicant pool they must start charging more.
(A) is apparently the answer, but all (A) states is that this explanation does in fact apply. It doesn't, however, state that none of the other possible explanations apply as well. Thus, even though charging more would increase the size of the applicant pool, it's not necessarily true that they must start charging more to increase the size of the applicant pool; other actions could produce this result as well.
(D) appears to me to be the correct answer. It states that there are no other explanations for the shrinking applicant pool. Thus, the one possible explanation given in the question stem must in fact be the correct explanation. Now, not only is it true that charging more will increase the size of the applicant pool, but it's also true that there's no other explanation and thus no other solution, e.g. they must charge more to increase the size of the applicant pool.
What am I missing?
The question stem states that one possible explanation for the shrinking applicant pool is charging too little. It also states that to increase the size of the applicant pool they must start charging more.
(A) is apparently the answer, but all (A) states is that this explanation does in fact apply. It doesn't, however, state that none of the other possible explanations apply as well. Thus, even though charging more would increase the size of the applicant pool, it's not necessarily true that they must start charging more to increase the size of the applicant pool; other actions could produce this result as well.
(D) appears to me to be the correct answer. It states that there are no other explanations for the shrinking applicant pool. Thus, the one possible explanation given in the question stem must in fact be the correct explanation. Now, not only is it true that charging more will increase the size of the applicant pool, but it's also true that there's no other explanation and thus no other solution, e.g. they must charge more to increase the size of the applicant pool.
What am I missing?